JonghyukJ247
Thanks Received: 0
Vinny Gambini
Vinny Gambini
 
Posts: 7
Joined: January 12th, 2023
 
 
 

Re: Q23 - Editorial: Given the law

by JonghyukJ247 Tue Feb 21, 2023 2:55 am

EricW539 Wrote:
ManhattanPrepLSAT1 Wrote:You're definitely down to the best two answer choices. At this point then, you want to find reasons to eliminate one of the two answer choices, rather than look for reasons to support one over the other.

Notice that towards the end of the first sentence, the argument says, "although other types of economies might be able to achieve [maximum total utility]."

That claim can be used to eliminate answer choice (C). The argument never says that it's the only way to bring about the end, just that it's the way most likely to achieve that end, so answer choice (D) is correct.

I hope that helps, let me know if you still have a hard time seeing this one!


How is D actually correct though?

The stimulus states that: if Max Utility is Guaranteed -> Pure Free Market

Yet D says they are trying to to bring a condition that ensures an end. Pure free market was never stated as sufficient condition, it cannot ensure anything. It only states in cases where max util is guaranteed, you must have a free market. A pure free market does not guarantee max util.

If it had said trying to bring about a condition required, I would agree.

How am I not understanding correctly?


I'm having the same question.

Pls help us!!!!